¿Por qué podemos considerar el punto final fijado en la derivación de la ecuación de Euler-Lagrange en mecánica?

En mecánica, obtenemos las ecuaciones de movimiento (ecuaciones de Euler-Lagrange) a través del principio de Hamilton considerando puntos estacionarios de la acción

S = t i t F L   d t
donde tenemos L = T V , la diferencia entre energía cinética y potencial. La derivación habitual pone a cero la primera variación e integra por partes, para dar el requisito
d S = t i t F [ L q d d t ( L q ˙ ) ] d q   d t + L q ˙ ( t F )   d q ( t F ) L q ˙ ( t i )   d q ( t i ) = 0
dónde q denota las coordenadas generalizadas y q ˙ las velocidades correspondientes.

En este punto, la mayoría de las derivaciones de libros de texto eliminan el segundo y el tercer término afirmando d q ( t i ) = 0 y d q ( t F ) = 0 . El primero de ellos es intuitivo, porque en la práctica normalmente consideramos problemas de valores iniciales en los que se conocen las posiciones iniciales. Pero, a priori, normalmente no sabemos q ( t F ) por un tiempo arbitrario t F , entonces, ¿por qué establecemos d q ( t F ) = 0 ?

Para algunos otros principios variacionales, es intuitivo suponer que las coordenadas en ambos extremos son conocidas y fijas, por ejemplo, el principio de Fermat para calcular la trayectoria de un rayo de luz entre dos puntos. ¿Existe una explicación intuitiva de por qué las coordenadas finales se consideran fijas cuando se aplica el principio de Hamilton, o una derivación de las ecuaciones mecánicas de Euler-Lagrange sin esta suposición?

Al considerar el problema yo mismo, traté de obtener las mismas condiciones de otra manera: si en cambio tomamos la posición final q ( t F ) como gratis pero con t F fijo, entonces, además de la ecuación de Euler-Lagrange, obtenemos el requisito adicional de estacionariedad

L q ˙ ( t F ) = 0
pero parece que esto no se cumple en general. Si consideramos un oscilador armónico, por ejemplo, esta condición implica que la energía cinética se minimiza en el tiempo fijo (arbitrario) t F . Todavía no he considerado las condiciones necesarias si también consideramos t F como libre, ya que no estoy totalmente seguro de cómo llevar a cabo el análisis sin incorporar elementos de la teoría del control óptimo (por ejemplo, el principio de Pontryagin o la ecuación HJB).

Gracias, no logré encontrar esas preguntas cuando busqué antes, y las respuestas a ellas son bastante interesantes. Sin embargo, no pude encontrar una derivación de las ecuaciones EL en aquellas donde el punto final se considera libre (que era la segunda parte de mi pregunta), aunque los comentarios en physics.stackexchange.com/questions/38348 mencionan que es posible.

Respuestas (2)

  1. Por lo general, en física, nos dan un problema, por ejemplo, un problema de valor inicial (IVP) o un problema de valor límite (BVP)? Estos dos tipos de problemas no deben confundirse, cf. por ejemplo , este , este y este Phys.SE publicaciones.

  2. En problemas dinámicos (a diferencia de los estáticos), un principio de acción estacionario o un principio de Maupertuis/principio de acción abreviado a veces son posibles para BVP, pero nunca para IVP si requerimos localidad 1 .

  3. Para el principio de acción estacionaria, existe cierta libertad matemática en la elección de condiciones de contorno (BC) consistentes, cf. por ejemplo, mi respuesta Math.SE aquí . Sin embargo, la física a menudo dicta qué BC son relevantes.

--

1 Existen varias formulaciones de acción no local para IVP, por ejemplo, el método bilocal de Gurtin-Tonti, cf. esta publicación Phys.SE.

¡Muchas gracias! Los puntos 1 y 3 son bastante claros, y el 1 en particular me hizo darme cuenta de que realmente estamos considerando un BVP cuando usamos el principio de Hamilton, pero las ecuaciones de movimiento son las mismas para los IVP del mismo sistema, por lo que aún podemos encontrar ellos usando Euler-Lagrange. ¿Tiene una referencia o alguna justificación para 2. ? Mi pensamiento original fue que tal vez sea posible una formulación variacional para un IVP al permitir que el tiempo final sea libre o infinito.
Tal vez estoy malinterpretando 2., pero si consideramos un IVP para el sistema de primer orden X ˙ = X con X ( 0 ) = X 0 , la solución viene dada por el minimizador de 0 ( X 2 + X ˙ 2 )   d t con X ( 0 ) = X 0 fijo, que podemos verificar introduciendo una variable de 'entrada' tu = X ˙ y aplicar el resultado del regulador cuadrático lineal estándar (LQR) de la teoría de control. ¿No es este un principio mínimo (local) para este IVP?

Si el principio de Fermat es intuitivo para usted, el principio de Hamilton no es muy diferente. Ambos básicamente afirman que el sistema (o la luz) se mueve entre dos puntos fijos de tal manera que la acción (o el tiempo) es máxima/mínima.

No estoy seguro de cuán intuitivo es esto. Probablemente, para la mayoría de las personas, lo más natural es pensar en el tiempo de desarrollo de los sistemas, es decir, preparar el sistema en algún estado (o enviar el haz de luz en cierta dirección) y ver qué sucede, es decir, donde termina.

Los principios de Hamiltion/Fermat son claros porque son generales (que es lo que les gusta a los físicos).

Con respecto a su pregunta, ¿por qué establecemos δq(tf)=0δq(tf)=0? , esto es básicamente lo que establece el principio de Hamilton. En otras palabras: observa todos los caminos posibles desde un punto inicial hasta un punto final y el sistema toma el camino ideal. No estás mirando todos los caminos posibles entre todo tipo de puntos.

Como ejemplo del mundo real, tome como punto de partida su hogar y como punto de destino su lugar de trabajo. Hay todo tipo de caminos que podrías tomar entre ellos. Sin embargo, usted (o la "naturaleza" si lo desea) decidirá por un solo camino, que es ideal. Dependiendo de sus prioridades (= acción funcional), este podría ser el camino que toma el tiempo más corto, o el camino que es el más barato, etc.

Tenga en cuenta que para encontrar este camino ideal no está considerando los caminos desde su casa hasta la piscina, o los caminos entre el trabajo y el aeropuerto, etc.

Gracias, su primer párrafo (junto con la respuesta de QMechanic) me hizo darme cuenta de que el principio de Hamilton realmente se refiere a un problema de valor límite, y que el problema de valor inicial que estaba considerando como más 'intuitivo' simplemente tiene la misma ecuación de movimiento, es decir, las ecuaciones de Euler-Lagrange.